PT 52 Sec 1 Q 10 Forum

Prepare for the LSAT or discuss it with others in this forum.
Post Reply
stcait

New
Posts: 46
Joined: Tue Oct 15, 2013 7:21 am

PT 52 Sec 1 Q 10

Post by stcait » Fri Jul 17, 2015 11:49 am

Hi,
I read all explanations on Mlsat, but I'm still confused with the stimilus and why A is correct...Could anyone explain A is the answer? :(
Thanks,

redfred22

New
Posts: 93
Joined: Fri Oct 10, 2014 9:26 pm

Re: PT 52 Sec 1 Q 10

Post by redfred22 » Fri Jul 17, 2015 12:29 pm

What exactly is tripping you up?

stcait

New
Posts: 46
Joined: Tue Oct 15, 2013 7:21 am

Re: PT 52 Sec 1 Q 10

Post by stcait » Fri Jul 17, 2015 12:49 pm

In the stimulus it says the payment will be enforced using the system, which will not be ready until the end of next year. But A says the charging will start before the end of next year...I thought those two contradict each other.
I thought they cannot charge without the system (bc the stimulus says so: the payment of this charge will be enforced using the system), but A sounds like saying the mayor is going to charge without using the system.
Not sure my question makes a sense, but I got so confused :(

redfred22

New
Posts: 93
Joined: Fri Oct 10, 2014 9:26 pm

Re: PT 52 Sec 1 Q 10

Post by redfred22 » Sat Jul 18, 2015 4:25 pm

Okay, so this questions has a couple layers, and it may be hard to explain through a simple text post.

Your last comment sort of answers your own question. The conclusion is that when it is first implemented, the plan will not be effectively enforced. We know that when the system is in place, it will be effectively enforced. The stimulus says, "without this system, however, mass evasion of the charge will result." The plan to charge the $10 and the system used to enforce it are two separate things. It's not one plan that will charge $10 and use the system.

So we know the conclusion. And it says that without the system, there will be mass evasion of the charge. Well, when the system is in place, we know that it will be effectively enforced. How could the author, right now, conclude that when it is first implemented that it will not be effectively enforced? The author would have to assume that the plan to charge will be implemented before the end of next year - when the system is put in place.

For necessary assumption questions, which this is, we can negate the right answer and it will destroy the argument. If we negate A, we get that the mayor's plan to charge will NOT be implemented before the end of next year. Well, if it's NOT implemented before the end of next year, that means it will be implemented when the system is available to use, and thus the claim that there will not be effective enforcement is destroyed.

stcait

New
Posts: 46
Joined: Tue Oct 15, 2013 7:21 am

Re: PT 52 Sec 1 Q 10

Post by stcait » Sun Jul 19, 2015 8:01 am

Thanks! :)
It seems I mistakenly think the plan and using the system must come together (bc it says "payment of this charge will be enforced using a highly sophiscated system") but as you say if they are two separate things that makes a sense.

Want to continue reading?

Register now to search topics and post comments!

Absolutely FREE!


Post Reply

Return to “LSAT Prep and Discussion Forum”